LSAT and Law School Admissions Forum

Get expert LSAT preparation and law school admissions advice from PowerScore Test Preparation.

 curiosity
  • Posts: 17
  • Joined: Jul 14, 2014
|
#15479
Why is answer choice C preferable over answer choice B? I had it narrowed down to the choice between the two and chose B. Thank you!
 Lucas Moreau
PowerScore Staff
  • PowerScore Staff
  • Posts: 216
  • Joined: Dec 13, 2012
|
#15484
Hello, curiosity,

The argument doesn't necessarily assume what's proposed in B. It doesn't presume that companies that pay the lowest wages have the lowest general operating expenses, only that their general operating expenses are "much lower than they would be" otherwise.

C is preferable because you can't achieve financial prosperity for your family by paying them a tiny wage, lol. :lol: But you probably already saw that! B just isn't as good as it might seem at first. :)

Hope that helps,
Lucas Moreau
 pacer
  • Posts: 57
  • Joined: Oct 20, 2014
|
#22427
Can you please explain this one? I really feel that B makes more sense since the authors fails to acknowledge any alternative reasons for operational cost levels. Choice C appears more like a distorted statement.
 Ron Gore
PowerScore Staff
  • PowerScore Staff
  • Posts: 220
  • Joined: May 15, 2013
|
#22428
Hi Pacer,

First, the portion of the argument that you're pointing to is a subconclusion of the argument, not it's main conclusion. You should always focus your prephrase on the argument's main point, unless you're specifically told otherwise. Second, answer choice (B) is not an accurate statement of the author's position. The author did not assume that the family businesses at issue pay the lowest wages, nor did the author assume that they have the highest profits. Rather, the author pointed to exceptionally low wages, which may or may not be the lowest wages, and to higher profits, which may or may not be the highest profits. Since this answer choice not an accurate reflection of the argument, it can't be correct.

I'm not sure why you think answer choice (C) is a "distorted" statement. Could you expand on that?

Ron
 pacer
  • Posts: 57
  • Joined: Oct 20, 2014
|
#22429
Hi Ron,

Thanks for the response.

For question 17, I understand now why B is incorrect. When I look at Choice C, it mentions family's prosperity (happiness etc) which implies to members of a family, focusing on individuals. But the stimulus is talking about financial prosperity. This made me think this was a distorted answer choice.

Thanks
 ksikanon
  • Posts: 13
  • Joined: Apr 14, 2018
|
#44984
Hi there,

A quick question on (B) - if the language was changed from "lowest" and "highest" to just "low" and "high," so that it reads "presumes, without providing justification, that businesses that pay low wages have low general operating expenses and thus high profits," would the answer be correct? Or has the stimulus provided us with premises that we can't question on this one?
 Daniel Stern
PowerScore Staff
  • PowerScore Staff
  • Posts: 81
  • Joined: Feb 07, 2018
|
#45010
I think your proposed linguistic change -- from "lowest" to "low" and "highest" to "high" -- still does not save answer choice B or make it a correct statement of the flaw in this argument. The author definitely does presume that lower wages lead to lower operating costs and thus higher profits, but I don't know that the author does so "without justification," as stated in answer B. Wages are, in fact, part of a business's operating cost, and lowering them, all else being equal, should increase profits.

When we are seeking a flaw, it is best to look at how the premises relate to the conclusion. Here, the premise that running a family business where the family members are paid low wages is supposed to lead to the conclusion that a family business will make the family financially prosperous. Credited response C points out the problem with this logic: that the low wages themselves may adversely effect the family's prosperity, regardless of the profitability of the business.

Good luck in your studies!
Dan
User avatar
 whitefox20
  • Posts: 6
  • Joined: Feb 06, 2021
|
#84983
Can someone please explain how we know that lower wages lead to lower operating costs? That is, how can we assume that all else is equal? Could it not be that some other expense in family businesses is higher and thus the general operating costs are higher as well? I just really don't see how we can safely assume this. I think this part really lead me to the wrong answer :(

Nevertheless, I do see why B is wrong now.

Could someone please also expand on what is the main flaw in this argument? As far as I understand, it is that higher profits do not necessarily guarantee surest road to financial prosperity...

Thanks for your help
User avatar
 KelseyWoods
PowerScore Staff
  • PowerScore Staff
  • Posts: 1079
  • Joined: Jun 26, 2013
|
#85073
Hi whitefox20!

To identify flaws in arguments, always start with identifying the premises and conclusions and then ask yourself why the premises don't fully prove the conclusion. So let's break down this argument:

Premise: In a family business, employees can be paid exceptionally low wages.
Sub-conclusion: Operating expenses are lower, which means profits are higher.
Conclusion: Family business is the surest way to family prosperity.

Now let's look for leaps between the premises and the conclusions.

The first leap is between the first premise and the sub-conclusion. Do lower wages prove that operating expenses are lower? Maybe not. Maybe there is some other operating expense that is specific to family businesses that compensates for the savings in wages. So, sure, maybe we can't necessarily assume that lower wages lead to lower operating costs. But I also can't think of another operating expense that would be specific to family businesses that other non-family businesses wouldn't also have. So this is possibly a flaw, but it doesn't seem like a very big one.

The second leap is between the sub-conclusion and the main conclusion. Do higher profits from lower wages necessarily mean greater family prosperity? Well now I've got a much bigger problem. Sure, the family is making more in profit, but they're making less in wages. So are they really better off with the higher profits than they would be with higher wages? It seems that, in terms of overall effect on family prosperity, it ends up being a bit of a wash. If I save $1000 because I'm paying my family lower wages than I would pay others and then end up with an extra $1000 in profits, is the family really more prosperous than it would be if I made $1000 less in profits but paid my family $1000 more in wages? This is a major flaw in the argument.

Arguments can have more than one flaw in them but in a Flaw in the Reasoning question, we only need an answer choice that describes one of the flaws, usually the biggest flaw. In this case, we don't have an answer choice that describes the assumption that lower wages automatically mean lower operating costs (answer choice (B) is too strongly worded to describe that flaw). But we do have an answer choice that describes the assumption that higher profits from lower wages are going to result in greater family prosperity. So that's the answer choice we have to go with.

Hope this helps!

Best,
Kelsey
User avatar
 whitefox20
  • Posts: 6
  • Joined: Feb 06, 2021
|
#85228
Thank you, Kelsey. This definitely makes sense. I agree that the flaw between sub-conclusion and the conclusion is much bigger. Somehow, I only focused more on the first flaw, which led me to answer choice B, which definitely is worded too strong. Thanks again for the explanation :)

Get the most out of your LSAT Prep Plus subscription.

Analyze and track your performance with our Testing and Analytics Package.